Math Is Fun Forum

  Discussion about math, puzzles, games and fun.   Useful symbols: ÷ × ½ √ ∞ ≠ ≤ ≥ ≈ ⇒ ± ∈ Δ θ ∴ ∑ ∫ • π ƒ -¹ ² ³ °

You are not logged in.

#1 Re: Jai Ganesh's Puzzles » Number Theory » 2009-04-30 19:08:07

M=10^5(m5)+10^4(m4)+10^3(m3)+10^2(m2)+10^1(m1)+10^0(m0) ≤ 10^5
⇒m5=0
⇒∑(10^i)mi ≤ 10^5 (i=1,2,3,4)...(1)
∑mi=10....(2) So the inequalility holds as max. value of M is 91000
∴No of M's satisfying (1)&(2) is = coefficient of x^10 in (x^0+x^1+x^2+.....upto ∞ terms )^5 - (5 c 4) (as mi can't take value 10;0≤mi≤10;0<x<1)
                                             =      ,,         ,,   ,,     ,, (1-x)^(-5) - (5 c 4)=(14 c 4) - 5=1001-5=996

Board footer

Powered by FluxBB